Docsity
Docsity

Prepare for your exams
Prepare for your exams

Study with the several resources on Docsity


Earn points to download
Earn points to download

Earn points by helping other students or get them with a premium plan


Guidelines and tips
Guidelines and tips

Porth’s Essentials of Pathophysiology 5th Edition Test Bank Latest Update 2023.Scored, Exams of Nursing

Porth’s Essentials of Pathophysiology 5th Edition Test Bank Latest Update 2023.Scored Grade A+.

Typology: Exams

2023/2024

Available from 12/11/2023

johnNice
johnNice 🇺🇸

268 documents

1 / 507

Toggle sidebar

Related documents


Partial preview of the text

Download Porth’s Essentials of Pathophysiology 5th Edition Test Bank Latest Update 2023.Scored and more Exams Nursing in PDF only on Docsity! 2 Porth’s Essentials of Pathophysiology 5th Edition Test Bank Latest Update 2023.Scored Grade A+ ,, Ch 1- Concepts of Health and Disease 1. At an international nursing conference, many discussions and breakout sessions focused on the World Health Organization (WHO) views on health. Of the following comments made by nurses during a discussion session, which statements would be considered a good representation of the WHO definition? Select all that apply. A) Interests in keeping the elderly population engaged in such activities as book reviews and word games during social time B) Increase in the number of chair aerobics classes provided in the skilled care facilities C) Interventions geared toward keeping the elderly population diagnosed with diabetes mellitus under tight blood glucose control by providing in-home cooking classes D) Providing transportation for renal dialysis patients to and from their hemodialysis sessions E)Providing handwashing teaching sessions to a group of young children Ans: A, B, C, E Feedback: The WHO definition of health is defined as “a state of complete physical, mental, and social well-being and not merely the absence of disease and infirmity.” Engaging in book reviews facilitates mental and social well-being; chair aerobics helps facilitate physical well-being; and assisting with tight control of diabetes helps with facilitating physical well-being even though the person has a chronic disease. Handwashing is vital in the prevention of disease and spread of germs. 2. A community health nurse is teaching a group of recent graduates about the large variety of factors that influence an individual's health or lack thereof. The nurse is referring to the Healthy People 2020 report from the U.S. Department of Health and Human Services as a teaching example. Of the following aspects discussed, which would be considered a determinant of health that is outside the focus of this report? A) The client has a diverse background by being of Asian and Native American descent and practices various alternative therapies to minimize effects of stress. B) The client has a family history of cardiovascular disease related to hypercholesterolemia and remains noncompliant with the treatment regime. C) The client has a good career with exceptional preventative health care benefits. P a g e 1 | 510 D) The client lives in an affluent, clean, suburban community with access to many health care facilities. Ans: B Feedback: In Healthy People 2020, the focus is to promote good health to all (such as using alternative therapies to minimize effects of stress); achieving health equity and promoting health for all (which includes having good health care benefits); and promoting good health (which includes living in a clean community with good access to health care). A client's noncompliance with treatments to control high cholesterol levels within the presence of a family history of CV disease does not meet the “attaining lives free of preventable disease and premature death” determinant. 3. A physician is providing care for a number of patients on a medical unit of a large, university hospital. The physician is discussing with a colleague the differentiation between diseases that are caused by abnormal molecules and diseases that cause disease. Which of the following patients most clearly demonstrates the consequences of molecules that cause disease? A) A 31-year-old woman with sickle cell anemia who is receiving a transfusion of packed red blood cells B) A 91-year-old woman who has experienced an ischemic stroke resulting from familial hypercholesterolemia C) A 19-year-old man with exacerbation of his cystic fibrosis requiring oxygen therapy and chest physiotherapy D) A 30-year-old homeless man who has Pneumocystis carinii pneumonia (PCP) and is HIV positive. Ans: D Feedback: PCP is an example of the effect of a molecule that directly contributes to disease. Sickle cell anemia, familial hypercholesterolemia, and cystic fibrosis are all examples of the effects of abnormal molecules. 4. A member of the health care team is researching the etiology and pathogenesis of a number of clients who are under his care in a hospital context. Which of the following aspects of clients' situations bNesUt RchSaIraNctGerTizBe.s pCaOthMogenesis rather than etiology? A) A client who has been exposed to the Mycobacterium tuberculosis bacterium B) A client who has increasing serum ammonia levels due to liver cirrhosis C) A client who was admitted with the effects of methyl alcohol poisoning D) A client with multiple skeletal injuries secondary to a motor vehicle accident Ans: B Feedback: B) High sensitivity; low specificity C) High specificity; low reliability D) High sensitivity; low reliability Ans: B Feedback: A large number of patients would receive the correct positive diagnosis (high sensitivity), while a significant number would receive a false-positive diagnosis (low specificity). The information given does not indicate low reliability or low validity. 10. As part of a screening program for prostate cancer, men at a senior citizens' center are having their blood levels of prostate-specific antigen (PSA) measured. Which of the following statements would best characterize a high positive predictive value but a low negative predictive value for this screening test? A) All of the men who had high PSA levels developed prostate cancer; several men who had low PSA levelNs UalRsoSdIeNveGloTpBe.d CprOostateM cancer. B) All of the men who had low PSA levels were cancer-free; several men who had high levels also remained free of prostate cancer. C) Men who had low PSA levels also displayed false-positive results for prostate cancer; men with high levels were often falsely diagnosed with prostate cancer. D) The test displayed low sensitivity but high specificity. Ans: A Feedback: The test's inability to rule out cancer with a low PSA level indicates a low negative predictive value. Answer B suggests a high negative predictive value, while answer C indicates a low positive predictive value. High positive predictive value is associated with high sensitivity. 11. A male international business traveler has returned from a trip to Indonesia. While there, he hired a prostitute for companionship and engaged in unprotected sex on more than one occasion. Unbeknownst to him, this prostitute harbored the hepatitis C virus. Upon return to the United States, he exhibited no symptoms and returned to his usual activities. During this period of no outward symptoms, the man would be classified as being in A) the preclinical stage of disease. B) remission and unlikely to develop hepatitis C. C) the clinical disease stage of hepatitis C. D) the chronic phase of hepatitis C. Ans: A Feedback: During the preclinical stage, the disease is not clinically evident but is destined to progress to clinical disease. 12. As of November 1, 2012, there were a total of 10 confirmed cases of Hantavirus infection in people who were recent visitors (mid-June to end of August, 2012) to Yosemite National Park. Three visitors with confirmed cases died. Health officials believe that 9 out of the 10 people with Hantavirus were exposed while staying in Curry Village in the Signature Tent Cabins. This is an example of A) what the anticipated mortality rate would be if a family of five were planning to vacation in Yosemite National Park. B) the prevalence of HantaNvUiruRsSoInNe GcaTnBa.ntCicOipMate if he or she is going to vacation in Yosemite National Park. C) the low rate of morbidity one can expect while traveling to Yosemite National Park. D) the incidence of people who are at risk for developing Hantavirus while staying in Yosemite National Park. Ans: D Feedback: The incidence reflects the number of new cases arising in a population at risk during a specified time. 13. A particular disease has a debilitating effect on the ability of sufferers to perform their activities of daily living and is a significant cause of decreased quality of life. However, few people die as a result of the disease's direct effects. There are hundreds of thousands of Americans living with the disease but relatively few new cases in recent years. Which of the following statements best conveys an accurate epidemiological characterization of the disease? A) Low mortality; high morbidity; low prevalence; high incidence B) Low mortality; high morbidity; high incidence; low prevalence C) High mortality; low morbidity; high incidence; low prevalence D) High morbidity; low mortality; high prevalence, low incidence Ans: D Feedback: Morbidity is associated with quality of life, while mortality is indicative of causation of death. In this case, morbidity is high and mortality is low. Prevalence refers to the number of cases present in a population, while incidence refers to the number of new cases. In this case, prevalence is high, while incidence is low. 14. An epidemiologist is conducting a program of research aimed at identifying factors associated with incidence and prevalence of congenital cardiac defects in infants. The researcher has recruited a large number of mothers whose infants were born with cardiac defects as well as mothers whose infants were born with healthy hearts. The researcher is comparing the nutritional habits of all the mothers while their babies were in utero. Which of the followiNnUg RtySpeIsNoGf TstBud.yCisOtMhe epidemiologist most likely conducting? A) Cohort study B) Cross-sectional study C) Case–control study D) Risk factor study Ans: C Feedback: In this study, the mothers with cardiac-affected babies would be the case group, while the mothers of healthy infants would serve as a control. This study does not possess the characteristics of a cohort or cross-sectional study, and risk factor study is not an existing methodology. 15. A nurse practitioner is working in a crowded neighborhood where the population is primarily immigrants from China. The nurse has designed a research study to follow children from kindergarten to the age of 25. She is going to be looking at their diet, successful progression in school, health practices, and development of disease, to name a few items. This type of research is known as A) cohort study. B) cross-sectional study. C) case–control study. D) epidemiological study. Ans: A Feedback: In this cohort study, a group of people who were born at approximately the same time or share some characteristics of interest is the focus of the research. This study does not possess the characteristics of a case–control or cross-sectional study, and epidemiological study is not an existing methodology. 16. As part of a community class, student nurses are developing a class to teach expectant parents the importance of having their child properly secured in a child safety seat. During the class, the students are going to have a safety officer examine the car seats that the parents have installed in their vehicles. This is an example of which type of prevention? A) Primary prevention B) Secondary prevention C) Tertiary prevention D) Prognosis enhancement Ans: A Feedback: Primary prevention is directed at keeping disease from occurring by removing risk factors. Some primary prevention is mandated by law, like child safety seats. Secondary prevention focuses on screening and early disease identification, whereas tertiary prevention is directed at interventions to prevent complications of a disease. 17. A multidisciplinary health care team operates a program aimed at the prevention, identification, and treatment of diabetes on a large Indian reservation. Which of the following aspects of the program would be most likely to be classified as secondary prevention? A) Regularly scheduled wound dressing changes for clients who have foot ulcers secondary to peripheral neuropathy and impaired wound healing 20. The neuroscience nursing unit has developed a set of step-by-step directions of what should occur if a nursing assessment reveals that the patient may be exhibiting clinical manifestations of a cerebrovascular accident (CVA). Which of the following statements about clinical practice guidelines are accurate? Select all that apply. A) Step-by-step guidelines are usually developed and based primarily on “how it has always been done before.” B) The development of evidence-based practice guidelines requires a research review from different studies to develop the most accurate diagnostic method to implement. C) Once developed, practice guidelines only need to be reviewed if a national committee sends out an update on new research. D) When developing a CVA set of step-by-step directions, the nursing unit should ask for assistance from experts in the neuroscience field. The potential users of the guidelines should pilot test it for further feedback. E)A meta-analysis could be utilized to combine evidence from different studies to produce a more accurate diagnostic method. Ans: B, D, E Feedback: Clinical practice guidelines are systematically developed and intended to inform practitioners in making decisions about health care for CVA patients. They should be developed using research and review by experts in the clinical content. Potential uses should also participate and provide feedback prior to implementation. The purpose of the guidelines is to review EBP articles and develop new practice guidelines rather than continuing practicing primariNlyUoRnS“hIoNwGiTt hBa.s CalOwMays been done before.”Once developed, the guidelines must be continually reviewed and changed to keep pace with new research findings. A meta-analysis could be utilized to combine evidence from different studies to produce a more accurate diagnostic method or the effects of an intervention method. Ch 2- Cell and Tissue Characteristics 1. During a discussion on cellular components and their function, a student asked the instructor the purpose of messenger RNA (mRNA). Of the following, which is the most accurate answer? A) Transports amino acids to the site of protein synthesis B) Acts as an inner nuclear support membrane for a rigid network of protein filaments that bind DNA to the nucleus C) Performs an active role of protein synthesis, where mRNA molecules direct the assembly of proteins on ribosomes to the cytoplasm D) Assists cells in forming neoplastic progression by altering the response of chromatin in the nuclear matrix Ans: C Feedback: The nucleus is the site for the synthesis of three types of RNA that move to the cytoplasm and carry out the actual synthesis of proteins. Messenger RNA copies and carries the DNA instructions for protein synthesis to the cytoplasm. Ribosomal RNA is the site of actual protein synthesis; transfer RNA transports amino acids to the site of protein synthesis. 2. The nurse is providing care for a client with a diagnosis of cirrhosis, and she notes that the client's sclerae are jaundiced. The nurse recalls that jaundice is a pigment that can accumulate in which part of the cell? A) Nucleus B) Cytoplasm C) Golgi apparatus D) Rough endoplasmic reticulum (ER) Ans: B Feedback: Pigments such as bilirubin and melanin can accumulate in the cytoplasm, resulting in the characteristic yellow skin tones associated with jaundice. Pigments do not tend to accumulate in the nucleus, Golgi apparatus, or rough ER. 3. A 14-year-old female has been experiencing severe internal cramps in the region of the pelvis and weight loss.She has been admitted with rectal bleeding.The physician has diagnosed her with inflammatory bowel disease (IBD). She asks the nurse what causes this disease. The nurse will base her response knowing that IBD has been linked to A) liver involvement in faulty glycogen stores. B) endoplasmic reticulum stress in the gastrointestinal system. C) oversecretion of insulin from the beta cells in the pancreas. D) infiltration of the gastrointestinal tract by bacterial toxins. Ans: B Feedback: Researchers are determining links between the endoplasmic reticulum (ER) and various disease processes. For examples, ER stress in the GI system has been found to be related to intestinal inflammations such as those occur with inflammatory bowel disease. The smooth ER of the liver is involved in glycogen stores. Insulin is synthesized as a large, inactive proinsulin molecule cut apart to produce a smaller, active insulin molecule within the Golgi complex of the beta cells. Bacterial toxins have exploited the retrograde transport mechanism. 4. A professor is teaching a group of students about the role of mitochondria within the cell. Which of the following statements is true of mitochondria? A) They are the site of adenosine triphosphate (ATP) production. B) The number of mitochondria in a cell is equal to the number of nuclei. C) They are replicated withNiUn RthSeIsmNoGoTthBe.nCdoOpMlasmic reticulum (ER). D) Mitochondrial DNA is inherited patrilineally. Ans: A Feedback: Consistent with their characterization as the “power plants” of the cell, mitochondria are the site of ATP synthesis for the cell. The number of mitochondria in a given cell type varies, according to the energy demands of the particular cell. They are self-replicating rather than being produced in the smooth ER, and they are inherited matrilineally. 5. A patient has been diagnosed with a neurodegenerative disease called multiple sclerosis (MS). The physician explains to the patient that this disease may be caused by dysregulated apoptosis. Later that day, the patient asks the nurse what this means. The nurse should reply, A) “The cells around your nerves don't know how to die correctly.” B) “The cytoplasm should neutralize the various apoptotic inhibitors but isn't working correctly.” C) “Dysregulated apoptosis has caused an excessive rate of programmed cell death along the neuropathways.” D) “There is an inappropriately low rate of apoptosis occurring within the cells.” Ans: C Feedback: Dysregulated apoptosis can mean too little or too much and has been implicated in neurodegenerative diseases, in which there is an increased or excessive rate of apoptosis. 6. A patient experiencing immotile cilia syndrome should be frequently assessed by the nurse for which priority complication? A) Epistaxis resulting from loss of cilia in the nasal passageway B) Bronchiectasis due to interferences with clearance of inhaled bacteria along the respiratory tract C) Sterility caused by inability of the sperm to swim downstream D) Inability to hear soft soNunUdRs SreIlaNteGdTtoBk.inCoOcMilium on the hair cells in the inner ear Ans: B Feedback: Immotile cilia syndrome immobilizes the cilia of the respiratory tract, thus interfering with clearance of inhaled bacteria, leading to the chronic lung disease called bronchiectasis. 7. A community health care worker is explaining to a group of factory workers the importance of wearing gloves when working with strong chemicals such as turpentine and paint thinner. Which of the following characteristics of cell membranes underlies the nurse's teaching? A) Cell membranes are impermeable to all but lipid-soluble substances. B) Cell membranes have a hydrophilic head and a hydrophobic tail. B) Facilitated diffusion C) Active transport D) Diffusion Ans: B Feedback: Facilitated diffusion involves the movement of a substance like glucose from an area of high concentration, such as the bloodstream, to an area of low concentration, such as the intracellular space, through the use of a transport protein. Osmosis, active transport, and diffusion do not bring about the movement of glucose into body cells. 13. An end-stage renal disease patient has been on peritoneal dialysis at home. Based on his lab work, he regulates the type of solution to infuse into his abdomen. When there is a high concentration of potassium inside the cell (hyperkalemia), the solution infused has a lower concentration so that the potassium ions will diffuse outward. At this point, the cellular membrane is said to be A) at equilibrium potential in which no net movement of ions occurs. B) charged with high voltage. C) filled with positive current. D) polarized because of the presence of a negative membrane potential. Ans: D Feedback: Because of the large concentration gradient existing across the cell membrane, potassium ions tend to diffuse outward. As they do so, they carry their positive charges with them, causing the inside to become negative in relation to the outside. This new potential difference repels further outward movement of the positively charged potassium ion. The membrane is said to be polarized. 14. A student asks the instructor about the origins of different tissues and their cellular origins during the process of development. Which of the instructor's following statements best describes the process of cell differentiation? A) “Cells of the hematopoietic system produce the appropriate body cells that are required at each stage of development.” B) “A single stem cell diffNerUenRtiSaIteNs iGnTtoBa.ppCrOoxMimately 200 different types of cells.” C) “A fertilized ovum undergoes a series of divisions, yielding many different cell types.” D) “Cells differentiate into necessary body cells peaking after conception and ceasing near the time of birth.” Ans: C Feedback: Cell differentiation and consequent tissue types are the outcome of the series of cell divisions that occur in the fertilized ovum. It originates neither with a single stem cell nor in the hematopoietic system. Stem cells allow for limited differentiation throughout the life span, not only antepartum. 15. A nurse is providing care for a client with a diagnosis of Crohn disease. The nurse recognizes the fact that the disease involves the inflammation and irritation of the intestinal lining. Which of the following types of tissue is most likely involved in the client's pathology? A) Simple columnar epithelium B) Glandular epithelium C) Simple cuboidal epithelium D) Stratified epithelium Ans: A Feedback: Simple columnar epithelium lines the intestine and has cilia and mucus-secreting goblet cells. The intestinal tract does not consist of glandular epithelium, simple cuboidal epithelium, or stratified epithelium. 16. During a crime scene investigation, the coroner confirms that rigor mortis has set in. This helps to confirm an approximate time of death. The forensic nurse can explain this process (rigor mortis) to a group of students based on the fact that A) troponin is being prevented from forming a cross-bridge between the actin and myosin. B) activated by ATP, cross-bridges become attached to the actin filament. C) the myosin head catalyzes the breakdown of ATP to provide the energy need so that a cross-bridge can be formed. D) at death, the body is unNabUleRtSoIcoNmGpTleBte.tCheOaMctin/myosin cycle and release the coupling between the myosin and actin, creating a state of muscular contration. Ans: D Feedback: As the muscle begins to degenerate after death, the sarcoplasmic cisternae release their calcium ions, which enable the myosin heads to combine with their sites on the actin molecule. As ATP supplies diminish, no energy source is available to start the normal interaction between the actin and myosin, and the muscle is in a state of rigor until further degeneration destroys the cross-bridges between the actin and myosin. 17. The homecare nurse is making a home visit to a 51-year-old female client with a long- standing diagnosis of multiple sclerosis. The nurse knows that the muscle wasting and weakness associated with the disease process are ultimately manifested as a failure of what normal process in muscle tissue? A) The contraction of the epimysium B) The surrounding of fascicles by perimysium C) Thick myosin and thin actin filaments sliding over each other D) The contraction of fascicles within myofibrils Ans: C Feedback: The contraction of skeletal muscle tissue can be characterized as the sliding action of myosin and actin. Epimysium surrounds and plays a supportive role in the skeletal muscle, and perimysium similarly provides support but does not actively produce locomotion. Myofibrils are found within fascicles, not the opposite. 18. An instructor is explaining to a group of students the way in which muscles and their associated tendons can be so strongly attached. The instructor makes references to the role of the basal lamina. Which of the following statements most accurately captures an aspect of the basal lamina? A) It is produced by the connective tissue adjacent to it. B) It is present where connective tissue contacts the tissue it supports. C) It is found solely between connective tissue and muscle fibers. D) It is also known as the bNaUseRmSeIntNmGeTmBb.raCnOe.M Ans: B Feedback: The basal lamina is found where connective tissue is in contact with the tissue that it provides support for. It is produced by epithelial cells and is found on Schwann cells, adipose tissue, and other sites apart from muscle fibers. It is not synonymous with the basement membrane, which consists of the combination of basal lamina and the reticular layer. 19. A caregiver is working with a client who is having poorly controlled pain due to shingles. The associate pain travels to the client's nervous system via A) synapses. B) axons. C) afferent neurons. D) efferent neurons. Ans: C Feedback: Afferent or sensory neurons carry information toward the CNS; they are involved in the reception of sensory information from the external environment and from within the body. Efferent or motor neurons carry information away from the CNS; they are needed for control of muscle fibers and endocrine and exocrine glands. D) possible rejection symptoms following a liver transplant. Ans: B Feedback: Hypertension is a common cause of adaptive hypertrophy, in which cardiac muscle cells increase in size in response to the increased work of circulation over time. The other diagnoses are not associated with muscle hypertrophy. 4. Which of the following clients is at a high risk for developing dilated cardiomyopathy? A) A 17-year-old with a diving injury resulting in paraplegia B) A 4-year-old child born with cerebral palsy and confined to a wheelchair C) A 44-year-old noncompliant female who forgets to take her hypertensive medications D) A 78-year-old patient with Alzheimer disease who received a third-degree burn following an oven fire Ans: C Feedback: In hypertension, the increasedNwUoRrSklIoaNdGrTeqBu.ireCdOtMo pump blood against an elevated arterial pressure in the aorta results in a progressive increase in LV muscle mass and need for coronary blood flow. The pressure overload causes hypertrophied cells to have greater width and length. Paraplegia, cerebral palsy, and Alzheimer disease do not increase the workload of the cardiac muscle per se. 5. Which of the following statements by a student demonstrates a sound understanding of the cellular processes of hypertrophy and hyperplasia? A) “I know that cells like neurons have little capacity for hyperplastic growth.” B) “A remaining kidney can sometimes undergo hyperplasia in response to one that has been removed.” C) “When male patients experience 'an enlarged prostate,' they are describing a form of hypertrophy.” D) “Clients with cardiomyopathy undergo myocardial hypertrophy with proportional increases in cell length and width.” Ans: A Feedback: Because they do not normally divide, neurons do not undergo hyperplasia. Remaining organs can display hypertrophy, not hyperplasia, and prostate enlargement is a form of hyperplasia. Cardiomyopathic hyperplasia involves greater increases in cell length than width. 6. A community health care nurse is teaching a group of female high school students about the importance of regular Papanicolaou (Pap) smears. The nurse recognizes that what fact underlies the rationale for this teaching? A) The active substitution of normal cells in the cervix correlates to cancer risk. B) Undifferentiated stem cells are an early indicator of cervical cancer. C) Cancer of the uterine cervix develops incrementally at a cellular level. D) Dysplasia in the connective tissue of the cervix is a strong precursor to cancer. Ans: C Feedback: Cervical cancer is indicated by incremental epithelial changes, beginning with dysplasia. Dysplasia does not involve active cellular substitution, as in the case of metaplasia, nor does it include a role for stem cells. Dysplasia does not normally occur in connective tissue, nor is the cervical lining made up of connective tissue. 7. A home health nurse is making a visit to a family with an 8-month-old infant with severe motor deterioration. The physician has diagnosed the infant with Tay-Sachs disease. The parents are asking the nurse why this happened. The nurse will base her answer knowing the root cause of Tay-Sachs is A) high exposure to lead in the home environment. B) an enzyme defect causing abnormal lipid accumulation in the brain. C) hypoxia caused by placing the infant on their abdomen during sleep. D) an increase in bilirubinNreUteRnStioInNlGeaTdBin.gCtoOdMestruction of RBCs. Ans: B Feedback: Tay-Sachs disease is a genetic disorder resulting from an enzyme defect that results in abnormal lipid accumulation in the brain and other tissues. Lead exposure, hypoxia, and bilirubin retention are not associated with Tay-Sachs disease. 8. A 68-year-old male client with aortic stenosis secondary to calcification of the aortic valve is receiving care. Which of the following statements best captures an aspect of this client's condition? A) Paget disease, cancer with metastases, or excess vitamin D may have contributed to the problem. B) Increased calcium intake over time may have contributed to the problem. C) The client has possibly undergone damage as a result of calcification following cellular injury. D) The client has possibly exhibited phosphate retention leading to calcium deposits. Ans: C Feedback: Dystrophic calcification is a result of deposition of calcium following cellular injury, such as that which commonly occurs in heart valves. Answers A, B, and D all refer to the phenomena associated with metastatic calcification and the associated increases in serum calcium levels. 9. A nurse in the emergency department admits a male client who has experienced severe frostbite to his hands and toes after becoming lost on a ski hill. The nurse recognizes that which of the following phenomena has contributed to his tissue damage? A) Decreased blood viscosity has resulted in interstitial bleeding. B) Reactive vasodilation has compromised perfusion. C) Autonomic nervous stimulation has resulted in injury. D) Decreased blood flow hNaUs iRnSduIcNedGhTyBpo.xCiaO. M Ans: D Feedback: Damage from exposure to cold results from hypoxia, ice crystal formation, and vasoconstriction. Blood viscosity increases, not decreases, and vasoconstriction occurs rather than vasodilation. 10. As part of a first aid class, a health care instructor is teaching a group of industrial workers about how electrical injuries can cause cell damage. Which of the statements made by one of the workers indicates that further teaching is necessary? A) “The greater the skin resistance, the greater the amount of deep and systemic damage a victim is likely to incur.” B) “The particular pathway that a current takes through the body is very significant.” C) “Resistance to flow is the phenomenon that transforms electrical energy into heat.” D) “The most severe damage is likely to occur where the current enters and leaves the body.” Ans: A Feedback: High skin resistance is associated with greater local and superficial burns, rather than deep and systemic damage. The pathway is indeed an important factor in the degree of injury, and this is a result of the transformation of current into heat. Damage is most severe at the point of entry and exit. A) permanent damage will occur in the myocardium if the vessel is not opened within a 1- to 2-minute window following the occlusion. B) treatment needs to be sought immediately so that the buildup of lactic acid is limited and cellular changes can be reversed. C) once the oxygen supply has been occluded, cellular changes are irreversible even if oxygenation is restored. D) the body will grow new genes through the process of angiogenesis, thereby avoiding any permanent damage to the myocardium. Ans: B Feedback: Ischemia is characterized by impaired oxygen delivery and impaired removal of metabolic end products such as lactic acid. Ischemia commonly affects blood flow through limited numbers of vessels and produces local tissue injury. In some instances, the cellular changes due to ischemia are reversible if oxygenation is restored. If not restored, permanent damage can occur. 17. Which of the following statements most accurately conveys an aspect of cell injury due to impaired calcium homeostasis? A) Normal intracellular caNlcUiuRmSiIonNlGevTeBls.aCreOhMigher than extracellular levels. B) Ischemia and certain toxins cause a decrease in cytosolic calcium. C) Injured cells tend to accumulate calcium. D) Low calcium levels cause an activation of damaging enzymes. Ans: C Feedback: Injured cells tend to accumulate calcium, though it is unclear whether this is evidence of causation of cell injury. Intracellular calcium levels are normally lower than extracellular levels, and ischemia and certain toxins cause an increase in cytosolic calcium. High calcium levels may cause an activation of damaging enzymes. 18. The nurse is providing care for a client with a diagnosis of amyotrophic lateral sclerosis (ALS). The nurse recognizes which of the following mechanisms is suspected to play a role in the cellular death associated with ALS? A) Apoptosis B) Liquefaction necrosis C) Hypoxic cell injury D) Caseous necrosis Ans: A Feedback: Although the exact mechanisms of damage are undetermined, ALS is thought to be caused by apoptosis. 19. The nurse is providing care for a 21-year-old female client with gas gangrene secondary to her compound fracture in her arm. Which of the following assessment findings would the nurse most reasonably expect to find when caring for a client with a diagnosis of gas gangrene? A) Inflammation of the affected tissue B) A positive culture for Staphylococcus C) Spreading edema D) Impaired alveolar gas exchange Ans: C Feedback: Spreading edema is a cardinaNl sUigRnSoIf NgaGsTgBan.gCreOneM. It is often caused by Clostridium bacteria, not Staphylococcus. Inflammation may exist at the interface between affected and unaffected tissue, but not in the dead, affected tissue. Impaired gas exchange would not be a sign. 20. Which of the following enzymes listed below is responsible for cancer cells' ability to prevent aging of the cells and contributes to cellular immortality that is so characteristic of this disease process? A) Oxidoreductase B) Telomerase C) Hydrolase D) Isomerase Ans: B Feedback: Some cells have telomerase, an enzyme that “rebuilds” telomeres and lessens or prevents shortening. Cancer cells have high levels of telomerase, which prevents senescence and contributes to the cellular immortality that characterizes cancer. Ch 4- Genetic Control of Cell Function and Inheritance 1. How could a health care professional most accurately explain an aspect of the underlying structure of DNA to a colleague who is unfamiliar with genetics? A) “DNA consists of nucleotides plus one of the four nitrogenous bases.” B) “In the base pairs, adenine combines with thymine and guanine with cytosine.” C) “Thymine and cytosine are considered the purine bases.” D) “The backbone of a DNA molecule consists of either deoxyribose or phosphoric acid.” Ans: B Feedback: The base pairings of DNA are such that adenine combines with thymine and guanine with cytosine. DNA also includes the sugar deoxyribose, while thymine and cytosine are considered the pyrimidine bases. The backbone of DNA includes both deoxyribose and phosphoric acid. 2. A student is trying to understand the possible reasons that a genetic abnormality might exist in an individual. Which of the following reasons is most plausible? A) DNA has combined with several types of protein and a small amount of RNA. B) Histones have exerted control on the folding of DNA strands. C) DNA has blocked genetic transcription by preventing access of nucleotides to the DNA surface. D) Chromatin has maintained its stable structure during the DNA replication process. Ans: D Feedback: To facilitate DNA replication and gene expression, chromatin must change its structure through the process of chromatin remodeling. Answers A, B, and C all denote normal genetic processes. 3. A 45-year-old client who experienced exposure to radiation during an industrial accident several years prior is being assessed. Which of the following phenomena may underlie the genetic changes that have been noted in the client? A) Base pairs may have been rearranged by the radiation in the accident. B) Endonucleases may have influenced the DNA structure following exposure. C) Two paired bases may have exchanged helical position after the accident. D) The radiation may have produced a redundant or degenerate genetic code. Ans: A Feedback: Page 1 that which of the following statements about the characteristics of human chromosomes is accurate? A) Individual variations are attributable to differences in appearance in autosomes. B) Chromosomes undergo variations during each episode of cell division. C) Autosomes contain the determination of an individual's sex. D) Each of the 22 pairs of autosomes has a homolog. Ans: D Feedback: Each of the 22 autosomes contains a homolog in the diploid cell. Autosomes are consistent from individual to individual with regard to appearance. Chromosomes retain their integrity in cell divisions, and sex determination is found in the 23rd chromosome. 9. A man and woman are eager to determine the sex of their unborn child and have asked the nurse at the fertility clinic how this is possible at an early stage of in vitro development. Which of the nurse’s responses best captures the genetic rationale for early sex identification through tissue samples? A) “The inactive X chromosome can be visible in a female.” B) “The cells of a male contain a Barr body that can be visualized.” C) “A normal female lacks Barr bodies.” D) “The number of visible Y chromosomes indicates the sex.” Ans: A Feedback: The fact that the inactive X chromosome can be visible as a Barr body in a female allows for the extrapolation of the number of X chromosomes and thus the sex associated with the cells. Normal male cells lack Barr bodies, but they are present in females. The number of X chromosomes ultimately determines the sex of an individual. 10. A health care researcher has identified the gene of interest in a particular genetic disorder as well as the gene's location Xq97. Where would one find a gene named Xq97? A) Band q, region 97 of the Y chromosome B) Band 7, region 9 of the short arm of the X chromosome C) Band 9, region 7 of the long arm of the X chromosome D) Band 9, region 7 of the short arm of the Y chromosome Ans: C Feedback: Page 4 In gene names, the first letter stands for the chromosome. The second indicates the arm of the chromosome, p (short) or q (long). The first numeral indicates the band, and the second one indicates the region within that band. 11. A child possesses a trait that is the result of the interaction of two different genes, neither of which could have produced the trait independently. Which of the following explanations best captures the genetic explanation for this? A) The trait is an expression of multiple alleles. B) Epistasis has dictated the phenotypic outcome. C) The phenomenon is an example of polygenic inheritance. D) The outcome is the result of the interaction between collaborative genes. Ans: D Feedback: The expression of two genes influencing the same phenotype, neither of which could have produced it alone, is an example of collaborative genes. Multiple alleles involve more than one gene at a particular locus affecting the same trait, and in epistasis, a gene masks the phenotypic effects of another nonallelic gene. Polygenic inheritance involves multiple genes each affecting a small influence on a genetic outcome. 12. A health care professional works in a context where there are a large number of clients who live with genetic disorders. Which of the following circumstances would most likely involve an individual who has a genetic disorder? A) The primordial germ cells of both of the individual's parents have undergone meiosis. B) Two chromosomes of the same number have been inherited from one parent. C) The individual possesses 22 pairs of autosomes. D) The individual's karyotype indicates separate X and Y chromosomes at chromosome 23. Ans: B Feedback: When two chromosomes of the same number are inherited from one parent, the result can be the disorder of uniparental disomy. Answers A, C, and D all relate normal genetic processes. 13. Mary is heterozygous for blue eyes, a recessive trait. John is homozygous for brown eyes, a dominant trait. What color eyes will their four children have? Page 5 A) Brown B) Blue C) Some will have blue, and some will have brown D) Impossible to tell Ans: A Feedback: A heterozygote with a dominant and a recessive allele will have the dominant phenotype. In Mary's case, this will manifest itself in her brown eyes. A homozygote with two dominant alleles will have the dominant phenotype, so John also has brown eyes. To have blue eyes, the children would have to inherit two alleles for blue eyes. Because they will inherit, at most, one recessive allele for blue eyes, the children's eyes will be brown. 14. Knowing that persons with blonde hair exhibit the phenotype of a recessive gene, which of the following genetic scenarios would most likely underlie such a trait? A) aa B) A heterozygous pairing C) Either AA or Aa D) Different alleles at a gene locus Ans: A Feedback: A recessive trait is expressed solely in a homozygous pairing, such as aa. A heterozygous pairing, in which there are two different alleles at a gene locus, will not express a recessive trait. Aa is an example of a heterozygous pairing. 15. A group of researchers have identified that the prevalence of two particular genetic disorders shares a statistical correlation. Which of the following statements best conveys the genetic rationale for this situation? A) There is likely a cause-and-effect relationship between the two genes responsible. B) The chromosomes containing each gene are likely closely situated. C) The genes causing each disorder are likely in the same section of the same chromosome. D) The disorders likely share the same locus. Ans: C Feedback: The genes causing these problems are likely proximate in the same chromosome. They would not likely be correlated if they were in different chromosomes, and the situation is not indicative of a Page 6 B) Haplotype mapping C) The human genome project D) Interference RNA (RNAi) Ans: D Feedback: Pharmaceutical companies are using RNAi to identify disease-related drug targets. There is considerable interest in harnessing RNAi for therapeutic purposes, linking the treatment of HIV infection and hepatitis C. Page 9 Ch 5- Genetic and Congenital Disorders 1. An infant who is 4 days postpartum has been diagnosed with a single-gene disorder. The parents of the child have a number of questions about the etiology of the health problem, which the physician is attempting to address in detail. Which of the following teaching points most accurately captures an aspect of single-gene congenital disorders? A) Affected genes are present on autosomal chromosomes rather than sex chromosomes. B) The majority of single-gene disorders manifest near the time of puberty. C) A particular defect can be caused by mutations at several different loci. D) Single-gene disorders are associated with existing rather than new mutations. Ans: C Feedback: A particular defect is not limited to any one specific locus. Single-gene disorders can be present on either autosomal or sex chromosomes, and they are primarily pediatric disorders. They can result from either existing or new mutations. 2. A male client of a nurse practitioner has an autosomal dominant disorder. The client and his partner are considering starting a family. Which of the following statements indicates the client has an adequate understanding of the genetic basis of this health problem? A) “I know there's no way of accurately determining the chance that my child will inherit the disease.” B) “My children who don't have the disease still run the risk of passing it on to their children.” C) “I know that new genetic mutations won't occur between generations.” D) “I know that a single mutant allele is to blame for the health problem.” Ans: D Feedback: Autosomal dominant disorders are the result of a single mutant allele from an affected parent. There is a 50% risk of transmission, and unaffected people do not pass on the disorder. New genetic mutations may occur between generations. 3. A 6-year-old girl with a diagnosis of Marfan syndrome is being assessed at a community health clinic. Which of the following assessments would be the health care professional's lowest priority? A) A test of the child's visual acuity B) A musculoskeletal assessment C) Tests of kidney function D) Cardiovascular assessment Ans: C Feedback: The autosomal dominant disorder of Marfan syndrome is primarily manifested in the ocular, cardiovascular, and skeletal systems. These assessments would take priority over renal assessment. 4. A clinician who works on a cardiac care unit of a hospital is providing care for a number of clients. Which client most likely has a genetic disorder arising from inheritance of a single gene? A) A short, thin, 56-year-old woman with hypertension B) A tall, thin, myopic, 28-year-old woman with mitral valve prolapse C) An overweight, middle-aged male smoker with coronary artery disease D) A thin, middle-aged nonsmoking man with a repaired atrial septal defect Ans: B Feedback: This patient exhibits several classic signs of Marfan syndrome, an autosomal dominant genetic disorder of the connective tissue. The other patients show signs of heart disease most likely caused by multiple factors. 5. The nurse working in a pediatric office is scheduled to assess a female adolescent diagnosed with neurofibromatosis (NF) type 1. During this assessment, the nurse should be assessing the teenager for which of the following clinical manifestations of NF-1? Select all that apply. A) Irregular menstrual periods B) Severe scoliosis C) Hearing loss D) Complaints of having a hard time concentrating in school E)Speech impediments Ans: B, D, E Feedback: NF-1 children have many problems, often not apparent until puberty. They tend to have large tumors that cause facial disfigurement, skeletal deformities such as scoliosis, and neurologic complications like learning disabilities, attention deficit disorders, and speech abnormalities. 6. A new older female client at a long-term care facility has a diagnosis of type 1 neurofibromatosis. As part of the intake assessment protocol for the facility, the clinical educator is teaching the care staff about the diagnosis. Which of the following statements most accurately conveys an aspect of neurofibromatosis? A) “The neurofibroma lesions are unsightly for the client, but they are not painful.” B) “Her diagnosis puts her at higher risk of developing a malignant neoplasm.” genetic material lost is small. Difficulty arises during meiosis. The chief clinical significance arises when the translocation carriers involve chromosome 21, which may produce a child with Down syndrome. 11. A physician is working with a 30-year-old male client with Down syndrome who has been admitted to hospital with a diagnosis of acute leukemia. Which of the following physical assessment findings would the physician be more likely to find in an examination of this client than in other clients without Down syndrome? A) Hepatomegaly B) Decreased visual acuity C) Congenital heart defects D) Diabetes mellitus Ans: C Feedback: Congenital heart defects are associated with Down syndrome. Hepatomegaly, visual disturbances, and diabetes are not associated with Down syndrome. 12. An 11-year-old girl is suspected of having Turner syndrome. Which of the following diagnostic tests would be the most useful component of screening to confirm or rule out the diagnosis? A) Computed tomography of the head B) Echocardiogram C) Bone scan D) Liver biopsy Ans: B Feedback: Because of the association beNtwUeRenScIoNngGeTnBita.l CheOaMrt defects and Turner syndrome, an echocardiogram would most likely yield useful results. Turner syndrome is not associated with brain, skeletal, or liver involvement, and these tests would be less likely to provide useful insight. 13. If a male child was born with Klinefelter syndrome, as the child matures and becomes an adolescent, the nurse will assess the child for which of the following clinical manifestations listed below? Select all that apply. A) Enlarged breast tissue B) Sparse facial and pubic hair C) Tall stature out of proportion D) Severe mental retardation E)Higher than average linguistic skills Ans: A, B, C Feedback: Klinefelter syndrome is characterized by breast enlargement, sparse facial and body hair, small testes, and inability to produce sperm. At puberty, testes do not respond to stimulation from gonadotropins and undergo degeneration. This leads to a tall stature with abnormal body proportions. Although the intellect usually is normal, most 47, XXY males have some language impairment. 14. As part of her prenatal care, a pregnant woman and her partner are being taught by a community health nurse. Which of the following points about the teratogenic effects of different substances should the nurse include in his teaching? A) “Your developing baby is most vulnerable during the first 2 months of your pregnancy.” B) “You need to be very careful with vitamin D and its derivatives.” C) “Keep in mind that a high percentage of genetic abnormalities are attributable to drug origins.” D) “Your best option is to avoid using any drugs during your pregnancy.” Ans: A Feedback: The period between days 15 and 60 is the most susceptible time during development. Vitamin A, not D, poses a particular risk, and only 2% to 3% of anomalies are attributable to drug and environmental factors. It is not necessary to categorically avoid medication but rather to use caution and heed FDA guidelines. 15. While taking a prenatal history, the nurse would be most concerned about severe teratogenic effects on the fetus if the mother admits to taking which medications prior to finding out that she was pregnant. Select all that apply. A) Warfarin (Coumadin) for chronic atrial fibrillation B) Ethyl alcohol ingestion regularly every weekend and some nights throughout the week C) Isotretinoin (Accutane)NfoUrRaScnIeNGTB.COM D) Over-the-counter cetirizine (Zyrtec) for seasonal allergies E)Tetracycline for acne Ans: A, B, C, D Feedback: Several medications have been considered teratogenic. They include thalidomide, antimetabolites, warfarin, anticonvulsants, ethyl alcohol, cocaine, propyulthiouracil, tetracycline, and Accutane. 16. A woman gives birth to a small infant with a malformed skull. The infant grows abnormally slowly and shows signs of substantial cognitive and intellectual deficits. The child also has facial abnormalities that become more striking as it develops. What might you expect to find in the mother's pregnancy history? A) Folic acid deficiency B) Chronic alcohol use C) Chronic cocaine use D) Active herpes simplex infection Ans: B Feedback: The infant's signs and symptoms are characteristic of fetal alcohol syndrome. Folic acid deficiency is associated with neural tube defects, such as anencephaly and spina bifida. Cocaine use is associated with some of the same signs and symptoms as alcohol use but does not produce the characteristic facial abnormalities of fetal alcohol syndrome. Herpes simplex infection, although it is associated with microcephaly, hydrocephalus, defects of the eye, and hearing problems, also does not produce characteristic facial abnormalities. 17. Which of the following pregnant women has most likely encountered the greatest increase in the risk that her child will have a fetal anomaly? A) A woman with diagnoses of syphilis and cirrhosis of the liver B) A woman who has herpes simplex and who has recently recovered from endocarditis C) A woman with chronic obstructive pulmonary syndrome and tuberculosis D) A woman with diagnoses of insulin-dependent diabetes mellitus and peripheral neuropathy Ans: B Feedback: Herpes is among the microorganisms most commonly responsible for fetal anomalies. Syphilis and tuberculosis infections are also implicated but to a lesser degree. The other listed diagnoses are not noted to be associated with fetal anomalies. 18. While traveling throughout Asia, a young couple was exposed to many cultural experiences. One day, they were standing in line, and the person in front of them was clearly displaying signs of illness and had a pink or light red rash on his face with itching. Their guide commented on a recent outbreak of rubella. Upon return to the United States, the couple found out they were pregnant. Upon arrival at the clinic, they are very concerned about their possible exposure to rubella. From this history, the nurse knows that this infant is at high risk for which of the following complications? Select all that apply. A) Blindness or cataracts B) Deafness C) Facial deformities like small palpebral fissures or thin vermillion border D) Short, flipper-like appendages E)Small outbreak of blisters around its eyes and mouth 2 weeks after delivery Ans: A, B Feedback: Ch 6- Neoplasia 1. Which of the following characteristics could apply to healthy somatic cells rather than cancerous cells? A) A high rate of mutation exists in the cells. B) The cells have a reduced tendency to cluster together. C) They remain viable and multiply without attachments to other cells and the extracellular matrix. D) The cells are unable to proliferate except by mitotic division. Ans: D Feedback: Mitotic division is the normal method of division that exists in the body. High rates of mutation, reduced adhesion, and loss of anchorage dependence are associated with cancer cells. 2. Following a biopsy, a 54-year-old man has been diagnosed as having a benign neoplastic tumor. Which of the following characteristics most likely applies to his tumor? A) The tumor is poorly approximated and has the potential to break loose. B) The tumor may secrete hormones or cytokines. C) The well-differentiated, neoplastic cells are clustered together in a single mass. D) It has a rapid rate of growth and can induce ischemia. Ans: C Feedback: Benign tumors are composedNoUf wReSllI-dNifGfeTrBen.tiCatOedM, neoplastic cells that resemble the cells of the tissues of origin and are characterized by a slow, progressive rate of growth that may come to a standstill or regress. They tend to exist in a single mass. Malignant tumors tend to be poorly differentiated, grow rapidly, secrete hormones or cytokines, and have the potential to break loose. 3. A 77-year-old male client with a diagnosis of stomach cancer has been found to have metastases in his liver. The client and his family are surprised at this turn of events, stating that they do not see how he could have developed cancer in his liver. Which of the following facts would underlie the reply that the care team provides? A) The parenchymal tissue of the liver is particularly susceptible to secondary malignancies. B) The portal circulatory system brings venous blood from the GI tract into the liver. Page 1 C) Hepatic stromal tissue shares characteristics with cancerous cells, including lack of anchorage dependence. D) The proximity of the liver to the stomach allows for direct spread of cancerous cells due to a lack of contact inhibition. Ans: B Feedback: Portal circulation brings venous blood into the portal vein of the liver, facilitating hematologic spread. The parenchyma of the liver possesses no particular susceptibility to cancer, and hepatic tissue does not share traits of cancerous cells such as low contact inhibition or a lack of anchorage dependence. 4. A 41-year-old female with a family history of breast cancer has had a baseline mammogram. She states that she performs monthly self-breast exams but really has a hard time evaluating her lumps since she has numerous cysts. At her annual mammogram, the technician views a suspicious area and refers her to the radiologist. She asks the nurse in the officNeU, “RHSoIwNcGanTaBl.uCmOp Mappear so quickly?” The nurse's response is based on which of the following principles? A) A tumor is undetectable until it has doubled 30 times and contains at least 1 billion cells. B) Many tumor cells never leave the M-phase of the cell cycle. C) Cancer cells are undifferentiated and come in various shapes and sizes. D) If the breast has a lot of cysts, then the fluid within those sacs makes it difficult to feel the hard lumps of a cancer. Ans: A Feedback: The ratio of dividing cells to resting cells in a tissue mass is called the growth fraction. The doubling time is the length of time it takes for the total mass of cells in a tumor to double. Tumors do not stay in the M-phase of the cell cycle. Undifferentiated cancer cells do come in various shapes and sizes, but this has nothing to do with the detection of the tumor by palpation. Breast cysts are fluid-filled sacs but are usually not cancerous. 5. Unbeknownst to her or her care team, a 51-year-old woman's breast cancer has an etiology rooted in the fact that tumor-suppressing genes are present but have been silenced. Consequently, she has not synthesized normal cancer-suppressing proteins, and neoplasia has resulted. What process has accounted for the woman's cancer? A) Chromosomal translocation Page 2 B) The “two-hit” hypothesis of carcinogenesis C) Epigenetic mechanisms D) A DNA repair defect Ans: C Feedback: Epigenetic mechanisms may silence genes, such as tumor suppressor genes, so that even though the gene is present, it is not expressed and a cancer-suppressing protein is not made. This process does not involve defects in DNA repair or chromosomal translocation, and while it may form a half of the “two-hit” hypothesis, this is not synonymous with epigenetic mechanisms. 6. Which of the following patients of a primary care physician would not require extra screening for cancer? A) A 51-year-old woman whose grandmother died of breast cancer B) A 48-year-old man who takes immunosuppressant drugs following a kidney transplant C) A 50-year-old male who is obese and has a low-fiber, high-fat diet D) A 38-year-old female wNitUhRDSoIwNn GsyTnBdr.oCmOe Mand congenital scoliosis Ans: D Feedback: While a family history of cancer, immunosuppression, and poor diet are all associated with cancer, congenital and chromosomal abnormalities are not noted to represent an increased risk for cancer. 7. Blood-borne cancerous cells have recently spread from a woman's primary tumor in her pancreas to her bones. Which of the following components of the woman's immune system are likely to be directly involved in the attempt to eradicate the potential metastasis? Select all that apply. A) T lymphocytes B) Macrophages C) Natural killer (NK) cells D) B cells E) Mast cells Ans: A, B, C, D Feedback: Virtually all of the components of the immune system have the potential for eradicating cancer cells, including T lymphocytes, B lymphocytes, antibodies, macrophages, and natural killer (NK) cells. Although best known for their role in allergy and anaphylaxis, mast cells play an important protective role as well, being intimately involved in wound healing and defense against pathogens. Page 3 B) Tumor markers C) Microarray technology D) Tissue biopsy Ans: A Feedback: Immunohistochemistry can be used to determine the site of origin of metastatic tumors. In cases in which the origin of the metastasis is obscure, immunochemical detection of tissue-specific or organ- specific antigens can often help to identify the tumor source. Tumor markers, microarray technology, and biopsy are less likely to aid in identifying the primary source. 13. Which target of both chemotherapy and radiation treatment accounts for adverse as well as therapeutic effects? A) Cell surface receptors B) Circulating hormone levels C) Blood vessels D) Rapidly proliferating cells Ans: D Feedback: Chemotherapy and radiation treatment both preferentially affect rapidly proliferating cells that include some normal body cells, such as epithelial and hair follicle cells, as well as cancer cells. 14. Upon entering the room of a 74-year-old client receiving brachytherapy for cervical cancer, you find the radiation implant and the position-holding device in the client's bed. What is the nurse's first best action? A) Assess the client's mental status. B) Use tongs to place the implant in the radiation container. C) Notify the physician and move the client to a different room. D) Don gloves and attempt to reposition the implant and positioning device. Ans: B Feedback: To minimize staff radiation exposure, the rules of time, distance, and shielding come into play. If a radioactive devNicUe RdiSslIodNgGesT,Bth.eCreOsMhould be a lead container and tongs in the room, so the device can be safely stored until the radiology department can dispose of of it safely. Page 6 15. A cancer patient has been prescribed 5-fluorouracil, an antimetabolite chemotherapy agent. This medication stops normal development and division by interrupting the S-phase of the cell cycle. When teaching this patient, the nurse explains that during the S-phase of the cell cycle, A) the cell is in a prolonged resting state and only leaves this state when cellular destruction is occurring. B) the DNA synthesis stops, but RNA synthesis continues. C) nuclear division occurs. D) the synthesis of DNA occurs, causing two separate sets of chromosomes to develop. Ans: D Feedback: During the S-phase, DNA synthesis occurs, causing two separate sets of chromosomes to develop. Antimetabolites can cause abnormal timing of DNA synthesis. Because of their S-phase specificity, the antimetabolites are more effective when given as a prolonged infusion. 16. A 61-year-old male client is scheduled to begin chemotherapy for the treatment of his bone cancer shortly. Staff at the cancer center have educated the man and his wife about the goals, course, and expectations of his treatment. Which of the following medications and treatments might the man anticipate needing during and after his course of treatment? A) Analgesia and corticosteroids B) Antiemetics and packed red blood cell (PRBC) transfusions C) Whole blood transfusion and antiplatelet aggregators D) Diuretics and selective serotonin reuptake inhibitors (SSRIs) Ans: B Feedback: Nausea and anemia are common side effects of chemotherapy and may be addressed with antiemetics and PRBCs. There is no noted indication with chemotherapy for corticosteroids, antiplatelet aggregators, diuretics, or SSRIs. 17. A patient with malignant melanoma has been prescribed alpha interferon, a biologic response modifier. Since this drug prolongs the cell cycle, increasing the percentage of cells in the G0 phase, and stimulates NK cells and T-lymphocyte killer cells, the nurse can anticipate that he may experience which of the following common side effects? A) Fever, chills, and fatigue B) Nausea, vomiting, and diarrhea C) Opportunistic infections like Candida Page 7 D) Renal damage with an iNnUcrReaSseIdNcGreTaBtin.inCeOlMevel Ans: A Feedback: Interferon is a biologic response modifier that changes a person's own immune response to cancer. This medication is given by injection, usually every other day. Because of stimulation of the body's natural immune response, the patient experiences extreme flulike symptoms. 18. An oncology nurse who has worked for many years providing care for children with cancer has taken a job on an adult oncology unit of a hospital. What differences might the nurse anticipate in this new job? A) There will be a greater number of cancers that are epithelial in origin. B) A greater proportion of the clients will have cancer that involves the hematopoietic system. C) The nurse will be working with more clients who have blastomas. D) More clients will be receiving treatment for leukemia. Ans: A Feedback: Epithelial cancers are more common in adults, while “blastomas” and cancers of the hematopoietic system such as leukemia are more common in children. 19. A 26-year-old man who survived childhood acute lymphocytic leukemia (ALL), one of the most common childhood cancers, now complains of weakness, fatigue, and shortness of breath. His treatment for ALL likely included anthracyclines. What is the most likely cause of his symptoms? A) Recurrence of ALL B) CNS problems resulting from childhood chemotherapy C) Heart failure resulting from childhood chemotherapy D) Hormonal dysfunction resulting from childhood chemotherapy Ans: C Feedback: The patient's symptoms resemble those of congestive heart failure. The anthracyclines, such as doxorubicin and daunorubicin, are associated with the risk for developing cardiomyopathy and heart failure. Page 8 A) The release of antidiuretic hormone (ADH) from the posterior pituitary gland B) Shivering in response to low environmental temperature C) The platelet aggregation mechanism for closing minute ruptures in very small blood vessels during accidental injury D) Increased production of white blood cells (WBCs) in response to a microorganism Ans: C Feedback: The release of hormones during labor increases rather than mitigates a physiological system. Specifically, uterine contraction stimulates the production of other relevant hormones that cause temporary instability that culminates in childbirth. ADH counters the potential instability of insufficient hydration and/or blood pressure, much as shivering is an attempt to counter low temperature. Increased production of WBCs is a response to the potential homeostatic instability of an infectious process. 4. An occupational health officer who works in the context of a large police force is attempting to understand the role that stress may play in the health of his clients. According to Selye, which of the following statements best captures an aspect of the phenomenon of stress? A) The alarm stage involves the release of cortisol and catecholamines. B) The nature of a stress response is determined by the objective severity of the stressor. C) Systemic illnesses can sometimes result from the resistance stage of stress response. D) Periods of stress can be developmentally positive or negative. Ans: D Feedback: Selye noted that stress can result in positive growth and development, and that stress is not unanimously detrimental to health and development. The alarm stage is associated with the release of cortisol and catecholamines, and a stress response is dependent on properties of the stressor and the individual's conditioning; the severity is not objectively determined. Illness is often a consequence of the exhaustion stage. 5. An 80-year-old woman is slated for total hip replacement the following day and is experiencing a large amount of stress around her potential surgical outcomes. Which of the following is most likely to be uninvolved in the physiological response to her stress? A) Hypothalamus B) Parathyroid C) Adrenal cortex D) Pituitary gland Ans: B Feedback: The physiology of stress is associated with the hypothalamic–pituitary–adrenal axis. The parathyroid is not noted to be centrally involved in stress response. 6. A 45-year-old woman with a diagnosis of shingles is experiencing an acute onset of severe neuropathic pain. Which stage of Selye's characterization of stress response is the woman most likely experiencing at the moment? A) General adaptation syndrome (GAS) B) Exhaustion C) Alarm D) Resistance Ans: C Feedback: The stimulation of the sympathetic nervous system, such as that during an episode of pain, characterizes the alarm NstaUgRe.SRIeNsiGstTanBc.e CanOdMexhaustion would likely follow, and GAS encompasses the whole continuum, not just this particular stage. 7. When explaining to a patient admitted for stress-induced supraventricular tachycardia, the nurse will incorporate which of the following statements about what happens in the body as a result of excess stress? A) The primary role of the parasympathetic nervous system is to stimulate the vagus nerve. B) The corticotropin-releasing factor stimulates the release of norepinephrine, which is responsible for “fight-or-flight” reaction to stress. C) Endorphins are released from the brain every time we experience stress. D) The pituitary gland is ultimately responsible for growth, metabolism, and maturation, all of which are important when the body is stressed. Ans: B Feedback: Norepinephrine stimulates the secretion of CRF, and CRF stimulates the release of norepinephrine. 8. A student is attempting to trace the feedback cycle involved in the stress response. Which of the following neural structures is thought to be the central integrating site for the stress response? A) Hypothalamus B) Cerebral cortex C) Locus ceruleus D) Reticular formation Ans: C Feedback: The locus ceruleus (LC) produces norepinephrine (NE), which initiates the autonomic syndrome known as “fight-or-flight.” The LC–NE system is connected to the hypothalamus, the limbic system, the hippocampus, and the cerebral cortex, which carry out the specific functions of the stress response. 9. A medical student is scheduled to undergo a clinical exam of his assessment skills under the supervision of the attending physician. As a result of this stressor, the student has high serum levels of glucocorticoid hormones such as cortisol. Which of the following statements best captures an aspect of the role of glucocorticoid hormones such as cortisol in the physiological response to stress? A) Blood glucose levels are increased in anticipation of energy expenditure. B) The immune system is bolstered in response to an impending threat. C) Osteoblast activity and protein synthesis are suppressed in order to refocus energy. D) Attention, arousal, and respiratory rate are increased in order to prepare for a response. Ans: C Feedback: Cortisol suppresses osteoblast activity, hematopoiesis, protein and collagen synthesis, and immune responses with the goal of preserving energy for a fight-or-flight response. Blood glucose levels are stabilized, not increased, and the immune system is not prioritized in the stress response. Increased attention, arousal, and respiratory rate are the domain of the locus ceruleus–norepinephrine system. 10. The physician suspects that a patient under a lot of stress (stimulation of the sympathetic nervous system) is experiencing stress-induced release of vasopressin. Knowing the effects of an activated renin–angiotensin–aldosterone system (RAAS), the nurse should be assessing the patient for which primary nursing diagnosis? A) Fluid volume excess related to retention of water in the body B) Decreased cardiac output related to positional low blood pressure C) Electrolyte imbalance related to retention of potassium not associated with sleep deprivations. With sleep disorders, the REM sleep cycle decreases, affecting creativity and forming associations. Restless leg syndrome is not associated with insomnia. 15. A patient with Parkinson disease has challenged himself to maintain mobility for longer than the physician predicts. He strives every day to walk 5 to 10 steps further than the day before. This phenomenonN, UbeRinSgIrNesGeaTrBch.eCd ObyMsocial psychologists, is known as A) stubbornness. B) assertiveness. C) hardiness. D) positivism. Ans: C Feedback: Hardiness describes a personality characteristic that includes a sense of having control over the environment, a sense of having a purpose in life, and an ability to conceptualize stressors as a challenge rather than a threat. 16. Which of the following patients, when faced with acute stressful situations, would be considered at highest risk for becoming noncompliant with their medication regimen? A) End-stage renal failure patient experiencing electrolyte imbalances related to having trouble sticking to his prescribed diet B) Teenager whose period is late by 2 weeks and afraid to tell her parents C) Paraplegic patient who forgot his pressure control device and has a small decubitus on his coccyx D) New mother who is breast-feeding every 2 to 3 hours throughout the day and night. Ans: A Feedback: For people with health problems (like ESRD patients), acute stress can interrupt compliance with medication regimens and exercise programs. 17. Based on the assessment results, which of the following hospital patients is most likely to be experiencing the effects of chronic stress? A) A 32-year-old intravenous drug user with a diagnosis of endocarditis B) A 45-year-old paraplegic who is experiencing delayed wound healing of an ischial ulcer C) A 79-year-old woman who has a diagnosis of acute and chronic renal failure D) A 63-year-old man with a diagnosis of chronic obstructive pulmonary disease (COPD) and heart failure (HF) Ans: B Feedback: Delayed wound healing has been associated with chronic stress. Endocarditis, renal failure, COPD, and CHF are not noted as common effects of chronic stress. 18. A counseling psychologist is working with a 30-year-old female client who is experiencing the symptoms of posttraumatic stress disorder (PTSD) following a house fire several months prior. Which of the client's following diagnostic results could most likely be interpreted as a manifestation of PTSD? A) The client has decreased levels of cortisol. B) The client has decreased levels of norepinephrine. C) The client has decreased levels of angiotensin II. D) The client has increased levels of growth hormones. Ans: A Feedback: Decreased cortisol levels are a marker of PTSD and a differentiation from other subtypes of stress. Low levels of norepinephrine and angiotensin II would not accompany any common variant of stress response, and growth hormones are suppressed in response to stress. 19. During a period of stress, the nurse asks the patient to close his eyes and think of a calm, relaxing place where he can feel the wind blowing on his cheek and smell the salty air from the ocean. This is an example of utilizing which type of treatment for stress reduction? A) Hypnosis B) Guided imagery C) Yoga D) Biofeedback Ans: B Feedback: Guided imagery is another technique that is used to achieve relaxation. One method is scene visualization, in which the person is asked to sit back, close the eyes, and concentrate on a scene narrated by the therapist. Whenever possible, all five senses are involved. 20. A health care professional has recommended biofeedback to a client as a method of dealing with the high levels of stress in her life. Which of the following explanations best characterizes an aspect of biofeedback treatment? A) “You might be asked to use an electrocardiogram as part of the therapy.” B) “The goal is to make you aware of your physiological processes.” C) “You'll hopefully be able to gain control over skeletal muscle contractions.” D) “You'll become aware of the increased skin temperature that accompanies anxiety.” Ans: C Feedback: The goal of biofeedback is to gain control over muscle contractions that accompany anxiety and tension. Electrocardiograms are not used in biofeedback, and control, rather than simply awareness, is the ultimate goal. Skin temperature decreases, not increases, with anxiety. Ch 8- Disorders of Fluid and Electrolyte, and Acid Base Balance 1. During a period of extreme excess fluid volume, a renal dialysis patient may be administered which type of IV solution to shrink the swollen cells by pulling water out of the cell? A) 0.9% sodium chloride B) 5% dextrose and water C) 3% sodium chloride D) Lactated Ringer solution Ans: C Feedback: When cells are placed in a hypotonic solution, which has a lower effective osmolality than the ICF, they swell as water moves into the cell, and when they are placed in a hypertonic solution, which has a greater effective osmolality than the ICF, they shrink as water is pulled out of the cell. 2. A 34-year-old male client has diagnoses of liver failure, ascites, and hepatic encephalopathy secondary to alcohol abuse. The client's family is questioning the care team as to why his abdomen is so large even though he is undernourished and emaciated. Which of the following statements most accurately underlies the explanation that a member of the care team would provide the family? A) An inordinate amount of interstitial fluid is accumulating his abdomen. B) The transcellular component of the intracellular fluid compartment contains far more fluid than normal. C) Normally small transceNlluUlRarSflIuNidGcTomB.paCrtOmMent, or third space, is becoming enlarged. D) Muscle cramps, pins and needle sensation around the mouth/lips, and unexplained bruising Ans: A Feedback: SIADH manifests as a dilutional hyponatremia. Decrease urine output, absence of edema, and headaches are signs of this. Answer choice B relates to s/s of diabetes insipidus; answer choice C is indicative of common flu s/s; answer choice D is relates to s/s of hypocalcemia. 8. A 77-year-old female hospital patient has contracted Clostridium difficile during her stay and is experiencing severe diarrhea. Which of the following statements best conveys a risk that this woman faces? A) She is susceptible to isotonic fluid volume deficit. B) She is prone to isotonicNfUluRidSvIoNluGmTeBex.cCesOs.M C) She could develop third-spacing edema as a result of plasma protein losses. D) She is at risk of compensatory fluid volume overload secondary to gastrointestinal water and electrolyte losses. Ans: A Feedback: This woman is at risk of isotonic fluid volume deficit and sodium imbalances as a result of her diarrhea. She is not likely to develop fluid volume excess or third spacing as consequences of diarrhea. 9. You are volunteering in the medical tent of a road race on a hot, humid day. A runner who has collapsed on the road is brought in with the following symptoms: sunken eyes, a body temperature of 100°F, and a complaint of dizziness while sitting to have his blood pressure taken (which subsides upon his lying down). These are signs of a fluid volume deficit. Which of the following treatments should be carried out first? A) Offer water by mouth. B) Begin cooling of his body by ice packs. C) Give him a transfusion of FFP. D) Give him an electrolyte solution by mouth. Ans: D Feedback: Fluid volume deficit results in postural hypotension (dizziness while upright) due to decreased blood volume. Sunken eyes and elevated temperature also point to a fluid volume deficit. The most important action to take is to replace fluid; however, pure water would be a mistake, since without accompanying electrolytes such as sodium, hyponatremia (water retention and a decrease in serum osmolality) could result. Thus, an oral electrolyte solution is recommended; in more severe cases, an IV would be appropriate. 10. A client is brought to the emergency department with complaints of shortness of breath. Assessment reveals a full, bounding pulse, severe edema, and audible crackles in lower lung fields bilaterally. What is the client's most likely diagnosis? A) Hyponatremia B) Fluid volume excess C) Electrolyte imbalance: hypocalcemia D) Hyperkalemia Ans: B Feedback: Peripheral and pulmonary edema as well as a bounding pulse and dyspnea are indicators of fluid volume overload. 11. A 26-year-old male patient with a diagnosis of schizophrenia has been admitted with suspected hyponatremia after consuming copious quantities of tap water. Given this diagnosis, what clinical manifestations and lab results should the nurse anticipate the patient will exhibit? A) High urine specific gravity, tachycardia, and a weak, thready pulse B) Low blood pressure, dry mouth, and increased urine osmolality C) Increased hematocrit and blood urea nitrogen and seizures D) Muscle weakness, lethargy, and headaches. Ans: D Feedback: Weakness, lethargy, and nausea are noted manifestations of hyponatremia. High urine specific gravity, tachycardia, and a weak, thread pulse are associated with hypernatremia, while low blood pressure, fever, and increased urine osmolality are manifestations of fluid volume deficit. Increased hematocrit and blood urea nitrogen and seizures are also associated with hypernatremia. 12. An ECG technician is performing an ECG on a hospital patient who has developed hypokalemia secondary to diuretic use. Which of the following manifestations of the client's health problem will the technician anticipate on the ECG? A) Irregular heart rate and a peaked T wave B) A low T wave and an absent P wave C) A prominent U wave and a flattened T wave D) A narrow QRS complexNaUnRdSaIn NabGsTenBt .UCwOaMve Ans: C Feedback: ECG changes associated with hypokalemia include a prominent U wave and a flattening of the T wave. Atrial fibrillation, a low P wave, and the absence of a U wave are not associated with hypokalemia. 13. A nurse in a medical unit has noted that a client’s potassium level is elevated at 6.1 mEq/L. The nurse has notified the physician, removed the banana from the client’s lunch tray, and is performing a focused assessment. When questioned by the client for the rationale for these actions, which of the following explanations is most appropriate? A) “Your potassium level is high, and so I need you let me know if you feel numbness, tingling, or weakness.” B) “Your potassium levels in the blood are higher than they should be, which brings a risk of changes in the brain function.” C) “I'll need to monitor you today for signs of high potassium; tell me if you feel as if your heart is beating quickly or irregularly.” D) “The amount of potassium in your blood is too high, but this can be resolved by changing the intravenous fluid you are receiving.” Ans: A Feedback: Paresthesia and muscle weakness are manifestations of hyperkalemia. Tachycardia and dysrhythmias are more commonly associated with hypokalemia, and the greatest risks associated with potassium imbalances are cardiac rather than neurological. Hyperkalemia is not normally resolved by correction using IV fluid. 14. A renal failure patient with severe hyperkalemia (K+ level 7.2 mEq/L) has just been admitted to the nursing unit. Given the severity of this situation, the nurse should be prepared to administer which intravenous infusion stat? A) Lactated Ringer solutioNnUatR1S5I0 NmGLT/hBou.rCtoOmaintM ain blood glucose levels B) Regular insulin infusion, rate dependent on lab values C) Infusion of Solu-Medrol to decrease irritation to the intravascular system D) Dilaudid via patient-controlled device (PCA) to control pain Ans: B Feedback: The administration of sodium bicarbonate, -adrenergic agonists, or insulin distributes potassium into the ICF compartment and rapidly decreases the ECF concentration. Lactated Ringer solution, steroids, or narcotics will not help to lower potassium levels. should assess the patient for which of the following clinical manifestations of hypocalcaemia? Select all that apply. A) Personality changes B) Hyperactive reflexes C) Increase in ventricular arrhythmias D) Increase in bouts of atrial fibrillation E)Symptomatic hypotension Ans: A, B, C Feedback: Hypocalcaemia may be evidenced by personality changes and neuromuscular irritability along with tremors, choreiform movements, and positive Chvostek or Trousseau signs. Cardiovascular manifestations include tachycardia, hypertension, and ventricular dysrhythmias. 21. A male patient with a history of heavy alcohol use has been admitted to hospital for malnutrition and suspected pancreatitis. The patient's diagnostic workup suggests alcoholic ketoacidosis as a component of his current health problems. He is somewhat familiar with the effect that drinking has had on his nutrition and pancreas but is wholly unfamiliar with the significance of acid–base balance. How best could his care provider explain the concept to him? A) “The chemical processes that take place throughout your body are thrown off very easily when your body is too acidic or not acidic enough. B) “The multitude of chemical reactions that take place in your body depend on your body fluids being slightly acidic.” C) “The healthy function of your kidneys and your lungs requires a specific level of pH in your body.” D) “Your body is highly dependent on what food and fluid you consume to keep itself at a functioning level of slight nonacidity.” Ans: A Feedback: Metabolic activity is highly contingent on a narrow range of pH. Normal pH is slightly basic, not acidic, and appropriate pH is maintained by the lungs and kidneys, not vice versa. The action of the respiratory and renal systems, not particular food or fluid intake, has the most salient effect on the acid–base balance. 22. When explaining how carbon dioxide combines with water to form carbonic acid as pa rt of acid–base lecture, the faculNtyUiRnsStrIucNtGorTeBm.pChaOsiMzed that which enzyme is needed as a catalyst for this reaction? A) Carbonic anhydrase B) Phenylalanie hdroxylase C) Hydrolases D) Trypsin Ans: A Feedback: Although CO2 is a gas and not an acid, a small percentage of the gas combines with water to form H2CO3. The reaction that generates H2CO3 from CO2 and water is catalyzed by an enzyme called carbonic anhydrase. 23. A 31-year-old client with a diagnosis of end-stage liver failure has been admitted to t he intensive care unit of a hospital. Arterial blood sampling indicates that the man has a n acid–base imbalance. Which of the following situations is most likely to result in an inappropriate pH? Page 1 A) Conservation or formation of new HCO 3 – by the kidneys B) Low albumin and plasma globulin levels C) Transcompartmental exchange of H+ a nd potassium ions D) Renal excretion of HCO 3 – in the presence of excess base Ans: B Feedback: Albumin and plasma globulins are key protein buffers in the vascular compartment; consequently, a low albumin level, as is common in liver failure, is apt to result in acid–base imbalances. Answer choices A, C, and D all convey normal physiological processes that help to maintain pH. 24. A patient with ESRD comes into the emergency department in severe acidosis. The nurse notes that the respiratory rate is 36 breaths/minute. The nurse unders tands the pathophysiology of this response and explains t o the student nurse that the patient's A) anxiety level is high, and the body i s trying t o release endorphins. B) chemoreceptors in the carotid and aortic bodies have noticed the pH change a nd altered the ventilator rate. C) kidneys are not able to buffer the acid and require the help from the lungs. D) lung s are trying to excreNteU eRxSceIsNsGhyTdBro.gCenO.M Ans: B Feedback: The second line of defense against acid–base disturbances is the control of extracell ular CO2 by the lungs. Blood PCO2 and pH are important regulators of venti lation. Chemoreceptors in the brain stem and the peripheral chemoreceptors in the carotid a nd aortic bodies sense changes in PCO2 and pH and alter the ventilatory rate. 25. Place the following stages of the hydrogen ion elimination and bicarbonate conservatio n in the proximal tubules of the nephrons in the ascending chronological order. Use all the options. A) CO2 and H2O are produced. B) H+ is secreting into the tubular fluid. C) Carbonic acid is produced. D) H+ combines with filtere d HCO 3 – . Ans: B, D, C, A Feedback: Renal regulation of pH involves the secretion of hydrogen ions into the tubular fluid, t he combining of hydrogen ions with bicarbonate yielding carbonic acid, followe d by the decomposition of carbonic acid into carbon dioxide and water. Page 2 ordinary, but an open antifreeze container was found in the boy's room. Which of the following is likely to be used to treat the patient's symptoms? A) Gastric lavage B) Syrup of ipecac C) Fomepizole D) Sodium bicarbonate Ans: C Feedback: Ethylene glycol is found in products ranging from antifreeze and deicing solutions t o carpet and fabric cleaners. It tastes sweet and is intoxicating—the factors that contribut e to its abuse potential. A lethal dose is approximately 100 mL. It is rapidly absorbe d from the intestine, making treatment with either gastric lavage or syrup of ipeca c ineffective. Fomepizole, with specific indications for ethylene glycol poisoning, wa s recently approved by the U.S. Food and Drug Administration. 32. A patient who overdosed on aspirin is brought to the emergency department. The nurse caring for this patient should anticipate which of the following clinical manifestations? Select all that apply. A) Respiratory rate of 40 B) BP 100/72 C) ABG report: pH 7.50, PNCUOR2S31INmGmTHBg., aCnOdMHCO3 level 19 mmol/L. D) Urine output approximately 100 mL/hour E) Bilateral crackles (fluid) in the lungs Ans: A, C Feedback: The salicylates cross the blood–brain barrier and directly stimulate the respira tory center, causing hyperventilation and respiratory alkalosis (answer choices A and C). The blood pressure is at normal range, and the urine output is normal or excessive depe nding on fluid intake. Bilateral crackles (fluid) in the lungs are usually a sign of heart failure. 33. Which of the following individuals are at risk of developing metabolic alkalosis? Selec t all that apply. A) A 70-year-old woman who has taken two tablespoons of baking soda to settle he r “sour stomach” B) A hospital patient who is on nasogastric suction following gastric surgery C) A 20-year-old male who has been regularly inducing himself to vomit follo wing binge eating Page 5 D) A 33-year-old male patient who is on mechanical ventilation in the intensive ca re unit following a head injury E) A 58-year-old alcoholic male who has been foregoing food for several wee ks while drinking heavily F) A 60-year-old female who has chronic renal failure secondary to hyperte nsion Ans: A, B, C Feedback: Ingestion of bicarbonate, gastric suction, and vomiting are causes of metabolic alkalosi s. Patients on mechanical ventilation are at risk of respiratory alkalosis, while hea vy alcohol use and renal failure are associated with acidosis. 34. A 55-year-old male client with a history of cardiovascular disease has been admitted to the intensive care unit after recovering from cardiogenic shock. In the hours sinc e admission, the client's arterial blood gases indicate acidosis, most likely acute lacti c acidosis. Which of the following signs, symptoms, and diagnostic findings might his care team anticipate before thNe UacRidS–IbaNsGe TbaBla.nCceOiMs restored? Select all that apply. A) Decreased pH B) Cardiac dysrhythmias C) Decreased alertness and cognition D) Hypoventilation E) Nausea and vomiting Ans: A, B, C, E Feedback: As with any form of acidosis, pH is apt to be lower than normal. Metabolic acidosis is also associated with dysrhythmias, decreased alertness, and nausea and vomiti ng. Respiration is likely to be increased in both rate and depth. 35. A nurse is providing care for a client who has been diagnosed with metabolic alkalosi s after several days of antacid use. Which of the following treatments should the nurse prepare to give? A) Intravenous or oral administration of free hydrogen ions B) Intravenous administration of KCl solution C) Administration of oxygen and NaHCO3 solution D) Supplementary oxygen and possible mechanical ventilation Ans: B Feedback: Page 6 KCl administration facilitates the renal retention of hydrogen ions, resulting in lowe ring of pH. It is not possible to administer free H+ ions, and sodium bicarbonate would exacerbate her condition. Mechanical ventilation is indicated in cases of respira tory acidosis. 36. A hospital patient's arterial blood gases indicate normal levels of oxygen and increase d carbon dioxide. The patient's respiratory rate is 12 breaths/minute (normal 1 4 to 20 breaths/minute) with all other vital signs within normal range. While not evident fr om assessment and diagnostics, the patient's kidneys are minimizing both H+ excretion a nd HCO 3 – reabsorption. What is this client's most likely diagnosis? A) Respiratory alkalosis B) Metabolic acidosis C) Respiratory acidosis D) Metabolic alkalosis Ans: D Feedback: In response to increased bicarbonate, the client is hypoventilating to increase ca rbon dioxide partial pressure. As well, renal compensation is aimed at lowering pH by both reducing H+ excretion and HCO3 – reabsorption. The given data are incongruent with the other major acid–base imbalances. 37. A 77-year-old female diagnosed with chronic obstructive pulmonary disease (COPD) is experiencing impaired gas exchange and CO2 retention, despite a rapid respiratory ra te. Which of the following pathophysiological principles would her health care team expec t if her compensatory mechanisms are working? A) Arterial blood gas sampling indicates a pH in the range of 7.45 to 7.55. B) Her kidneys are likely to reabsorb H+ and secrete HCO 3– C) Her body will produce excess metabolic CO2. D) Her kidneys will adapt with an increase in plasma HCO 3 – and her pH will decrease. Ans: D Feedback: Respiratory acidosis is accompanied by renal adaptation with a more marked increase in plasma HCO 3 – and a lesser decrease in pH. Her pH is likely below 7.35, and the likely renal response involves the reabsorption of HCO 3 – and secretion of H+. Excess CO 2 production is not a common manifestation of obstructive lung disease. 38. The ICU nurse is concerned with her patient's arterial blood gas (ABG) results— especially the pH 7.30; and PCO2 49 mm Hg. The nurse interprets these ABG result s to mean respiratory acidosis. The nurse knows which of the following are clinica l manifestations of respiratory acidosis? Select all that apply. A) Headache with complaints of blurred vision Page 7 A) Migration and proliferation of mast cells, neutrophils, and platelets to the injury site occupy an increased volume of tissue. B) Potent vasodilation increases the total volume of vascular space at the site of inflammation. C) Osmotic flow of plasma into the intravascular space causes increased blood volume and interstitial fluid. D) Loss of plasma proteins causes an increase in interstitial osmotic pressure. Ans: D Feedback: Swelling is the result of plasma proteins leaving the interstitial space, resulting in increased osmotic pressure of interstitial fluid and movement of fluid into tissues. Blood components, vasodilation, nor increased intravascular volume accounts for swelling. 4. Which of the following phenomena best accounts for the increased presence of leukocytes at the site of inflammation? A) Existing leukocytes stick to the epithelial cells and move along blood vessel walls. B) Increased numbers of leukocytes are released into circulation via cytokine stimulation. C) Leukocytes are osmotically drawn from circulation into the interstitial space as a result of swelling. D) Epithelium expresses leukocyte stimulation factors in response to cell injury. Ans: A Feedback: During inflammation, leukocytes accumulate at the point of epithelial contact in the processes of margination, adhesion, and transmigration. This is not directly achieved by a way of increased leukocyte production or release, nor by osmotic pressure. The epithelium does not produce leukocyte stimulation factors. 5. When explaining the final stages of the inflammatory response to pathogens, the nurse will educate the patient about A) how the body can kill the pathogen by generating toxic oxygen and nitrogen products producing such things as nitric oxide and hydrogen peroxide. B) margination, which is a process whereby white cells (leukocytes) stick to the endothelium and accumulate along the blood vessel. C) the increase in vascular permeability, which lets fluids leak into the extravascular tissues. D) the promotion of tissue regeneration whereby monocytes and macrophages produce potent prostaglandins and leukotrienes. Ans: A Feedback: The latter stages of phagocytosis results in intracellular killing of pathogens accomplished by several mechanisms, including toxic oxygen and nitrogen products, lysozymes, proteases, and defensins. The metabolic burst pathways generate toxic oxygen and nitrogen products (i.e., nitric oxide, hydrogen peroxide, and hypochlorous acid). Margination is one of the early stages of the inflammatory response. Vascular changes occur with inflammation but are prior to the final stage. Macrophages arrive within hours at the inflammation site. 6. A deficiency in which of the following would result in an inhibition of the inflammatory response? A) Histamine B) Helper T cells C) B cells D) Vitamin K Ans: A Feedback: Histamine is a key mediator in the inflammatory system, unlike helper T cells, B cells, or vitamin K. 7. When educating a patient with a wound that is not healing, the nurse should stress which of the following dietary modifications to ward off some of the negative manifestations that can occur with inflammation? A) Increase the amount of calcium in the diet, especially drinking milk and eating cheese. B) This is the one time whereby you should eat more fat (both polyunsaturated and saturated), so you can absorb more fat soluble vitamins. C) Since there is a loss of plasma proteins, you should increase your intake of organ meats like liver. D) Increase your intake of oily fish and fish oil so that you will increase absorption of omega-3 polyunsaturated fatty acids. Ans: D Feedback: Dietary modification of the inflammatory response through the use of omega-3 polyunsaturated fatty acids, specifically eicosapentaenoic acid and docosahexaenoic acid, which are present in oily fish and fish oil, may be effective in preventing some negative manifestations of inflammation. 8. Tumor necrosis factor-a and IL-1 are major cytokines that mediate inflammation. If the patient is developing a systemic response to an infection, the nurse will likely assess which of the following clinical manifestations? Select all that apply. A) Elevated temperature B) Hypertension C) Tachycardia D) Decrease in urine output E)Anorexia Ans: A, C, E Feedback: IL-1 and TNF-a are mediators of the acute-phase responses associated with infection or injury. Features of these systemic responses include fever (elevated temperature), hypotension, tachycardia (increased heart rate), anorexia, increase in neutrophil count, and increased levels of corticosteroid hormones. 9. A 63-year-old woman has begun a diet that purports to minimize the quantity and effects of free radicals in her body. What physiological processes could best underlie her care provider's teaching about her new diet? A) Free radicals act as direct mediators in the inflammatory process. B) Free radicals inhibit the inflammatory response, limiting preadaptive response to infection. C) Free radicals increase cytokine expression and adhesion molecule levels, resulting in increased inflammation. D) Free radicals contribute to atherosclerosis and decreased immune response. Ans: C Feedback: Free radicals are thought to bring about an inappropriate inflammatory response by increasing cytokines and numbers of adhesion molecule. They are not direct mediators of inflammation and are not associated with decreased immune response but rather inappropriate inflammation. Free radicals are not associated with inhibition of the inflammatory response. 15. A hospital patient has a large, superficial wound on her elbow that was the result of shearing action when she was moved up in her bed. The patient's husband mentions that the wound looks infected and irritated because the wound bed is completely red. Which of the following responses would be inappropriate? A) “Even though it is red, it doesn't mean that the wound is infected.” B) “The red areas show that there is enough circulation to facilitate healing.” C) “Those are fresh blood vessels that are a sign of healthy healing.” D) “A thin sheet of blood clotting is actually desirable and not a sign that your wife's wound is infected.” Ans: D Feedback: Granulation tissue indicates sufficient circulation and angiogenesis associated with healthy wound healing. Granulation tissue consists of new blood vessels, not clotted cellular components. 16. Which of the following processes would most likely be considered an anomaly during the cellular phase of inflammation? A) Platelet aggregation B) Vasoconstriction C) Migration of phagocytic white cells D) Macrophage activity Ans: B Feedback: While vasoconstriction is a component of the immediate inflammatory response, the later cellular phase of inflammation is accompanied by vasodilation. Platelet aggregation, vasoconstriction, migration of phagocytic white cells, and macrophage activity are all associated with the cellular phase. 17. A patient who is recovering from burn injuries is discussing his prognosis with a physician. Which of the following teaching points about expectations for healing should the physician include? A) “Once your healing is complete, your skin will be just as strong as before your accident.” B) “You may find that the scar is a bit smaller than the area of the wound.” C) “You'll find that your new tissue is more elastic and fragile than the rest of your skin.” D) “The final remodeling phase of healing may last up to 3 months in your case.” Ans: B Feedback: Scars are often smaller than the original area of the wound. There is nearly always an accompanying loss of strength and elasticity, and remodeling can take over 6 months. 18. Of the following list of patients, who would likely benefit the most from hyperbaric oxygen therapy? A) A trauma patient who developed Clostridium spp., an anaerobic bacterial infection in his femur B) A patient who developed a fistula between her bowel and vagina following cervical cancer surgery C) A school-aged child who fell on gravel and has terrible road rash D) A football player who has torn a meniscus in his knee for the third time this year Ans: A Feedback: Hyperbaric oxygen is a treatment that has demonstrated improved wound healing in multiple types of injuries. It enhances wound healing by a number of mechanisms, including the increased killing of bacteria by neutrophils, impaired growth of anaerobic bacteria, and the promotion of angiogenesis. 19. Which of the following patients is most likely to have impairments to the wound- healing process? A patient with A) chronic obstructive pulmonary disease. B) a diagnosis of multiple sclerosis and consequent impaired mobility. C) poorly controlled blood sugars with small blood vessel disease. D) congenital heart defects and anemia. Ans: C Feedback: Diabetes mellitus is strongly associated with impaired wound healing. The other noted pathologies are less causative of deficiencies in the healing process. 20. A 79-year-old female resident of an assisted living facility receives care from a community nurse on a regular basis for treatment of a chronic venous leg ulcer. Which of the following factors would the nurse be most justified in ruling out as a contributing factor to the client's impaired wound healing? A) A lower skin collagen content than in younger adults B) Decreased fibroblast synthesis C) Slow reepithelialization D) Decreased antibody levels Ans: D Feedback: Older adults do not normally have diminished antibody levels. Low collagen levels, decreased fibroblast activity, and slow reepithelialization are common impediments to wound healing in the elderly. autonomic, nervous system will be in control of the sweati ng process. Because the air temperature exceeds his body temperature, he will not be a ble to lose heat in this way. 26. A health educator is teaching a group of colleagues about the physiology o f thermoregulation. Which of the following statements is most accurate? A) “Endogenous pyrogens induce host cells to produce exogenous pyrogens.” B) “Prostaglandin E2 (PGE2) exerts a direct fever-producing effect on the hypothalamus.” C) “PGE2 induces Kupffer cells to initiate a fever response via hepatic sinusoids.” D) “Arachidonic acid induces cytokines to act on the temperature regulation center .” Ans: B Feedback: PGE2 is the protein that exerts control on the hypothalamus and induces feve r. Exogenous pyrogens induce host cells to produce endogenous pyrogens, and Kupffe r cells produce PGE2. Cytokines do not act directly in the hypothalamus. 27. A physician is noting the recent vital signs for several patients on an acute medical wa rd of a hospital. Which of the following hospital patients with noninfectious diagnose s would most likely have a fever? A) A 71-year-old female with limited mobility, chronic obstructive pulmona ry disease, and vascular dementia B) A 33-year-old female with a postoperative deep vein thrombosis and pulmona ry embolism C) A 51-year-old obese male with hepatic encephalopathy secondary to alcoh ol abuse D) A 71-year-old male with congestive heart failure and peripheral edema Ans: B Feedback: Pulmonary emboli can produce a fever. The other diagnoses do not. 28. A 54-year-old man presents with a temperature of 38.8°C (101.8°F), a racing hea rt, fatigue, and an upset stomach after spending an afternoon building a deck on a very hot, humid day. The physician assessing the man is performing a differential diagnosis a s part of her assessment. Which of the following findings would suggest fever rather tha n hyperthermia as a cause of the elevation in the man's temperature? A) Moist skin B) Dizziness Page 3 C) Shivering D) Cognitive changes Ans: C Feedback: Shivering is a response to a signal for increased heat production. It would only occur on a hot day if the set point for temperature regulation were increased, as in the case o f fever but not hyperthermia. Moist skin, cognitive changes, and dizziness can occur with either fever or hyperthermia. 29. A nurse is providing care for a 44-year-old male client who is admitted with a diagnosi s of fever of unknown origin (FUO). Which of the following characteristics of the clie nt's history is most likely to have a bearing on his current diagnosis? A) The client is cachexic and an African American. B) The client is HIV positive and homeless. C) The client is malnourished, hypomagnesemic, and hypocalcemic. D) The client is receiving iNntUraRvSenIoNusGnToBrm.aClOsaMline with 20 mEq KCl. Ans: B Feedback: FUO is associated with HIV. The other aspects of the client's circumstances are not noted to correlate with FUO. 30. In the hospital setting, one of the best ways to lower a hyperthermic patient's feve r would be to facilitate conduction of heat from the body by A) providing frequent sponge baths with cool water. B) taking all covers/clothing off and pouring alcohol on the skin. C) placing him or her on a cooling mattress that circulates a coolant solution th rough the mattress. D) placing IV solutions into the freezer for 30 minutes prior to hanging them. Ans: C Feedback: Cooling mattress facilitates the conduction of heat from the body into the c oolant solution that circulates through the mattress. Care must be taken so that the cooli ng method does not produce vasoconstriction and shivering. Sponge baths and a lcohol solutions increase evaporative heat loss but may cool them too quickly. IV soluti ons should not be placed in a freezer. 31. A child aged 33 days is presented to the emergency department of a hospital by he r parents following a 2-day fever. Her temperature is 38°C (100.4°F) tympanicall y. Which of the following diagnostic tests is most clearly indicated? A) Electrolytes, blood urea nitrogen (BUN), and creatinine levels Page 4 B) Abdominal ultrasound C) Computed tomography (CT) of the head D) Urine for culture and sensitivity Ans: D Feedback: Infants with a fever are at risk of urinary tract infections, which would be diagnose d through a urine test for culture and sensitivity. Electrolytes, BUN and creati nine, CT head, and abdominal ultrasound are not as closely associated with differential diagnosis of the child's fever. 32. An 88-year-old resident of a long-term care home has been suffering from a 3-day onse t of increased shortness of breath and decreased oxygen saturation. At the hospita l, an anterior-posterior chest x-ray and sputum culture and sensitivity have confir med a diagnosis of bacterial pneumonia, yet the client's tympanic temperature has not exceeded 37.3°C (99.2°F). The health care team would recognize that whic h of the following phenomena likely underlies this situation? A) An older adult is often insensitive to exogenous pyrogens. B) An older adult is sometimes incapable of vasodilation. C) An older adult's hypothNalUamRuSsIhNasGdTimiB.niCshOeMd thermoregulatory ability. D) Infections manifest by cognitive changes in older adults. Ans: C Feedback: The hypothalamus in older adults is often less capable of thermoregulation than in younger clients. There are sometimes alterations in the release of endogenous pyroge ns and deficits in vasoconstriction. While infections do often manifest with cognit ive changes in older adults, this does not explain why fever is precluded. 33. A 14-year-old boy is participating in his school's track meet; the outdoor temperature is 99°F, and a teacher has found the boy sitting restless in the shade and disoriente d to time. The teacher notes that the student has dry skin in spite of the high temperature a nd the fact that he has recently completed a running event. The teacher calls for the sc hool nurse, who will recognize which of the following potential diagnoses and anticipate d hospital treatments? A) Heat exhaustion, likely treated with rest, shelter from the sun, and salt tablets B) Heat stroke, likely treated with submersion in cold water C) Heat stroke, likely treated with rehydration by intravenous hypotonic solution D) Heat exhaustion, likely treated with oral rehydration with cool water Ans: B Feedback: Page 5 39. A 38-year-old male client with mild hypothermia following a prolonged hike in the ra in is brought to hospital by ambulance. Which of the following sets of vital signs would b e the most characteristic of the client's diagnosis? A) BP 178/102; RR 12; HR 58 B) BP 109/68; RR 9; HR 130 C) BP 160/99; RR 30; HR 66 D) BP 138/84; RR 28; HRN11U1RSINGTB.COM Ans: D Feedback: Mild hypothermia is associated with accelerated HR, slightly increased BP, a nd hyperventilation; these parameters are best characterized by the vital signs referred in answer D. 40. Following a cardiothoracic surgery, where controlled therapeutic hypothermia wa s utilized to decrease metabolic demands, the nurse responsible for recovering this patie nt should be assessing for which of the following potential complications to c old cardioplegia? A) Development of a first-degree AV block B) Vasoconstriction resulting in weak pedal pulses, requiring the use of a Doppler C) Frequent premature ventricular contractions (PVCs) on the telemetry monitor D) Cyanosis in lower extremities with no blanching in the toes Ans: C Feedback: Potential complications to cold cardioplegia include such signs and symptoms a s ventricular dysrhythmias, decreased cerebral blood flow, and postoperative myocardia l depression. First- degree AV block is common in patients with heart proble ms. Vasoconstriction of vessels is an expected effect of cold cardioplegia along with c old, bluish lower extremities. Page 8 Ch 10- Mechanisms of Infectious Disease 1. Staphylococcus aureus commonly found in the skin, nares, and other body sites of patients without any signs and symptoms of infection is known as which of the following conditions listed below? A) An opportunistic infection B) A parasitic infestation C) Bacterial colonization D) A saprophytic infection Ans: C Feedback: The ongoing presence of bacteria in the body that do not cause infection is called colonization. Opportunistic infection by ordinarily nonpathogenic organisms can occur in immunosuppressed hosts. Parasitism is a condition in which the organism exists at the expense of, and without benefiting, the host. Saprophytes do not cause infection. 2. Which of the following statements is an accurate descriptor of the role of viruses in human infections? A) Viruses have no genetic material of their own. B) Some viruses are capable of transforming normal host cells into malignant cells. C) Viruses are often implicated in cases of transmissible neurodegenerative diseases. D) Viruses require stimulation after a latent period before they are able to produce symptoms. Ans: B Feedback: Viruses that are categorized as oncogenic are able to induce malignancy in host cells. Viruses have limited genetic material (either RNA or DNA), but no virus lacks genetic material. Transmissible neurodegenerative disease is associated with prions. Not all viruses include a latent period. 3. Which of the following types of pneumonia listed below is best characterized by an infective agent that produces sputum samples with a peptidoglycan cell wall, expresses endotoxins, replicates readily in broth and on agar, grows in clusters, has pili, and does not stain when exposed to crystal violet? A) Chlamydial B) Viral C) Mycoplasmal D) Bacterial Ans: D Feedback: Although chlamydiae, viruses, and mycoplasmas all can cause pneumonia, only bacteria have all of these characteristics. Chlamydiae and viruses are obligate intracellular organisms and therefore would grow only in cell culture, and mycoplasmas lack the peptidoglycan cell wall typical of bacteria. 4. The nurse will most likely assess which of the following clinical manifestations in a client who was diagnosed with Creutzfeldt-Jakob disease? A) Change in behavior and memory and loss of coordination leading to encephalopathy B) Gastrointestinal problems like vomiting and diarrhea C) Muscle inflammation and edema, making movements very painful D) Projectile vomiting, hypertension, and drowsiness caused by elevated ICP Ans: A Feedback: Creutzfeldt-Jakob disease is collectively called transmissible neurodegenerative disease that is characterized by a slowly progressive, noninflammatory neuronal degeneration and leads to a loss of coordination, dementia, and death. With this disease, encephalopathy is the primary presenting factor. GI problems, muscle inflammation, and ICP are not clinical manifestations associated with this prion. 5. The spirochete leptospires is primarily transmitted to farmers by A) an airborne mechanism. B) exposure to spores in the environment. C) direct contact with infected animals. D) a mosquito bite. Ans: C Feedback: The pathogenic leptospires inNfeUctRaSwIiNdeGvTaBri.etCy OofMwild and domestic animals. Infected animals shed the organisms into the environment through the urinary tract. Transmission to humans occurs by contact with infected animals or urine-contaminated surroundings. This spirochete is not transmitted by air, spores, or a fomite. 6. The nurse knows which of the following statements best describes the characteristic trait of risckettsiae related to Rocky Mountain spotted fever? Rickettsiae A) are eukaryotic. B) have both RNA and DNA. C) have a distinct spiral-shaped morphology. D) are neither gram negative nor gram positive. Ans: B Feedback: Rickettsiae combine characteristics of bacteria and viruses. Rickettsiae are prokaryotic but not spiral shaped. Rickettsiae are able to be classified by Gram stain like other microorganisms. 7. An 81-year-old female patient in a subacute medical unit of a hospital has developed an oral Candida albicans infection. Which of the following phenomena would the patient's nurse suspect as a key contributing factor to her infection?